View Single Post
  #4  
Old Friday, August 02, 2013
salehasirajulhaqch's Avatar
salehasirajulhaqch salehasirajulhaqch is offline
Junior Member
 
Join Date: Aug 2012
Location: Bannu
Posts: 5
Thanks: 1
Thanked 6 Times in 2 Posts
salehasirajulhaqch is on a distinguished road
Default Analytical Reasoning Question.. Lets Solve this question.

A university library budget committee must reduce exactly five of eight areas of expenditure — G, L, M, N, P, R, S, and W — in accordance with the following conditions:

If both G and S are reduced, W is also reduced.

If N is reduced, neither R nor S is reduced.

If P is reduced, L is not reduced.

Of the three areas L, M, and R, exactly two are reduced.

Question 1

If both M and R are reduced, which one of the following is a pair of areas neither of which could be reduced?

A. G, L

B. G, N

C. L, N

D. L, P

E. P, S
__________________
“Live as if you were to die tomorrow. Learn as if you were to live forever.”
Reply With Quote